Definite intergration area under curve bounded with line

Click For Summary
SUMMARY

The discussion centers on calculating the exact area under the curve defined by the equation y = x³ - 8x² + 20x, bounded by the x-axis and a vertical line through point B. The participants identified the stationary points A and B, with coordinates calculated as x = 10/3 and x = 2. The integration of the polynomial was performed, yielding the expression (4x³/4) - (8x³/3) + 10x², and the area of region R was determined by adding the area of a triangle to the standard integral of the polynomial.

PREREQUISITES
  • Understanding of polynomial functions and their properties
  • Knowledge of definite integration techniques
  • Familiarity with calculating areas under curves
  • Ability to analyze geometric shapes such as triangles
NEXT STEPS
  • Study the process of finding stationary points in polynomial functions
  • Learn about definite integrals and their applications in area calculations
  • Explore methods for calculating areas of geometric shapes, particularly triangles
  • Practice integration of polynomials with varying limits
USEFUL FOR

Students studying calculus, mathematics educators, and anyone interested in mastering integration techniques and area calculations under curves.

thomas49th
Messages
645
Reaction score
0

Homework Statement



A cruve has the equation y = x{3} - 8x^{2} + 20x. The curve has stationary points A and B. There is a line through B parallel to y-axis and meets the x-axis at the point N. The region R is bounded by the curve , the x-axis and the line from A to N. Find the exact area under the curve

Homework Equations


The Attempt at a Solution



Well I found the x co-ords of A and B, which is \frac{10}{3} or 2. I intergrated the curve and got

\frac{4x^{3}}{4} - \frac{8x^{3}}{3}+10x^{2}

no +C as we'll be having limits i presume

But i don't know how to get the area of region R... as there is a stupid line in the way!

Can somebody show/help me to do it.

Thanks :)
 
Physics news on Phys.org
Hey

My first advice is to a picture of your problem. After that you notice that the exercise is to calculate the integral from x=A to x=B of f, i.e. integration of a polynomial. I expect you know how to do that.
 
i can intergrate a polynominal easily and in the question paper there is a picture of the question. But because of this AN line, it's thrown me. How would you go about doing it.

Thanks
 
Hi thomas! :smile:

If I've understood the question right, all you have to do is add a triangle (whose area is obvious), and you get the standard integral. :smile:
 
ahhh i see cheerz :)
 
Question: A clock's minute hand has length 4 and its hour hand has length 3. What is the distance between the tips at the moment when it is increasing most rapidly?(Putnam Exam Question) Answer: Making assumption that both the hands moves at constant angular velocities, the answer is ## \sqrt{7} .## But don't you think this assumption is somewhat doubtful and wrong?

Similar threads

Replies
5
Views
2K
  • · Replies 7 ·
Replies
7
Views
2K
  • · Replies 12 ·
Replies
12
Views
2K
  • · Replies 10 ·
Replies
10
Views
2K
Replies
3
Views
2K
  • · Replies 2 ·
Replies
2
Views
2K
  • · Replies 13 ·
Replies
13
Views
5K
  • · Replies 59 ·
2
Replies
59
Views
5K
Replies
4
Views
2K
  • · Replies 2 ·
Replies
2
Views
2K